You are on page 1of 9
IIT-JAM | TIFR | JEST | CSIR NET | GATE | CUET Physics PrepKit Angular momentum, Torque & Rotational Equilibrium Section 1 | Angular Momentum and Torque Question 1.1 (a),(d) ‘The dimensions of the quantities in one (or more) of the following pairs are the same. Identify the pair (s). (a) Torque and work (b) Angular momentum and work (© Energy and Young’s modulus (@) Light year and wavelength Question 1.2 (c) ‘The time dependence of the position of a particle of mass m=2 is given by r = 2ti — 3¢’j. Its angular momentum, with respect to the origin, at time ¢ = 2 is a) 36k by - 34k -i) - 48k a) 48(i +) Question 1.3 (b) A particle of mass mis projected with a velocity vmaking an angle of 45° with the horizontal. The magnitude of the angular momentum of the projectile about the point of projection when the particle is at its maximum height his a) zero b) mv*/(4/29) ©) mv /(/29) a) magn? Question 1.4 (b) A mass m is moving with a constant velocity along a line parallel to the x-axis, away from the origin. Its angular momentum with respect to the origin a)zero._b) remains constant ©) goes on increasing d) goes on decreasing Question 1.5 (a) A particle undergoes uniform circular motion. About which point on the plane of the circle, will the angular momentum of the particle remain conserved ? (a) Centre of circle (b) On the circumference of the circle (© Inside the circle (@) Outside the circle Question 1.6 (4) A particle moves in a circular path with decreasing speed. Choose the correct statement. Visit: wwwmyprepkitin or Download the PrepKit App Page 1 Contact us: contact@myprepkitin Whatsapp: 9971029358 IIT-JAM | TIFR | JEST | CSIR NET | GATE | CUET Physics PrepKit (a) Angular momentum remains constant (b) Acceleration (a) is towards the centre (©) Particle moves in a spiral path with decreasing radius (d) The direction of angular momentum remains constant Question 1.7 (b) An equilateral triangle ABC formed from a uniform wire has two small identical beads initially located at A. The triangle is set rotating about the vertical axis AO. Then the beads are released from rest simultaneously and allowed to slide down, one along ABand other along ACas shown. Neglecting frictional effects, the quantities that are conserved as beads slides down are (a) angular velocity and total energy (kinetic and potential) (b) total angular momentum and total energy (c) angular velocity and moment of inertia about the axis of rotation (d) total angular momentum and moment of inertia about the axis of rotation Question 1.8 (¢) Abob of mass m attached to an inextensible string of length /is suspended from a vertical support. The bob rotates in a horizontal circle with an angular speed w rad/s about the vertical support. About the point of suspension (a) angular momentum is conserved. (b) angular momentum changes in magnitude but not in direction (c) angular momentum changes in direction but not in magnitude (d) angular momentum changes both in direction and magnitude Question 1.9 (mv, °/2\/29) A particle of mass mis projected at time t= 0 from a point Pon the ground with a speed v0, at an angle of 45° to the horizontal. Find the magnitude and direction of the angular momentum of the particle about Pat time t = v,/g. Question 1.10 (d) a A particle of mass 20 g is released with an initial velocity 5 m/s along the curve from the point A, as shown in the figure. The point A is at height h from point B. The particle slides along the frictionless surface. When the particle reaches point B, its angular momentum about Owill be Visit: wwwmyprepkitin or Download the PrepKit App Page 2 Contact us: contact@myprepkitin Whatsapp: 9971029358 IIT-JAM | TIFR | JEST | CSIR NET | GATE | CUET Physics PrepKit (Take, g= 10 m/s*) a) 8kg m’/s b) 3kg m’/s O2kg m/s d) 6 kg m/s Question 1.11 (©) A small mass mis attached to a massless string whose other end is fixed at Pas shown in the figure. The mass is undergoing circular motion in the x- y plane with centre at Oand constant angular speed «. Ifthe angular momentum of the system, calculated about Oand Pare denoted by L., and L,, respectively, then a) Land L., do not vary with time b) L, varies with time while L, remains constant with time ©) L, remains constant while L,, vs 4) L, and L, both vary with time Question 1.12 (a,b,c) The torque t on a body about a given point is found to be equal to A x L, where Aisa constant vector and Z is the angular momentum of the body about that point. From t follows that di 2 (a) GE is perpendicular to L atall instants of time dt (b) the component of L, in the direction of A does not change with time (© the magnitude of L does not change with time (@L does not change with time Question 1.13 (a,c) Consider a body of mass 1.0 kg at rest at the origin at time t = 0.A force F = (ati + Bj) is applied on the body, where @ = 1.0 N/s and = 1, ON. The torque acting on the body about the origin at time ¢ = 2. 0s is t, Which of the following statements is (are) true? a) |r| = 1/3Nm ction of the unit vector + k b) The torque tis in the 6) The velocity of the body att = 1sis v = (1 + 2/)m/s d) The magnitude of displacement of the body at t = 1s is 1/6 m. Visit: wwwmyprepkitin or Download the PrepKit App Page 3 Contact us: contact@myprepkitin Whatsapp: 9971029358 IIT-JAM | TIFR | JEST | CSIR NET | GATE | CUET Physics PrepKit Question 1.14 [JEE Adv. 2016] (a,b,d) The position vector r of particles of mass m is given by the following equation 7) = at’ + Be’j where a = 10/3 m/s?, B = 5m/s’andm = 0.1kg. Att = 1s, which of the following statement(s) is/are true about the particle? a) the velocity vis given by v = 10 + 10j m/s b) The angular momentum L with respect to the origin is given by L =~ 5/3k Nms ©) The force F is given by F = (i + 2j)N 4) The torque t with respect to the origin is given by t =— 20/3 k Nm Question 1.15 (a,c,d) Arigid uniform bar AB of length Lis slipping from its vertical position on a frictionless floor (as shown in the figure). At some instant of time, the angle made by the bar with the vertical is 8. Which of the following statements about its motion is/are correct? (a) Instantaneous torque about the point in contact with the floor is proportional to sin 0 (b) The trajectory of the point Ais parabola (©) The mid-point of the bar will fall vertically downward (d) When the bar makes an angle 8 with the vertical, the displacement of its mid-point from the initial position is proportional to (1— cos®) Question 1.16 [JEE Adv 2017] (a,c) Awheel of radius Rand mass Mis placed at the bottom of a fixed step of height Ras shown in the figure. A constant force is continuously applied on the surface of the wheel so that it just climbs the step without slipping, Consider the torque t about an axis normal to the plane of the paper passing through the point Q. Which of the following options is/are correct? (a) Ifthe force is applied normal to the circumference at point P, then tis zero (b) If the force is applied tangentially at point S, then t #0 but the wheel never climbs the step (c) Ifthe force is applied at point Ptangentially, then t decreases continuously as the wheel climbs Visit: wwwmyprepkitin or Download the PrepKit App Page 4 Contact us: contact@myprepkitin Whatsapp: 9971029358 IIT-JAM | TIFR | JEST | CSIR NET | GATE | CUET Physics PrepKit (@) If the force is applied normal to the circumference at point X, then tis constant Question 1.17 (4) ‘Two thin circular discs of mass mand 4m, having radii of a and 2a, respectively, are rigidly fixed by a massless, rigid rod of length | = (24a through their centres. This assembly is laid ona firm and flat surface and set rolling without slipping on the surface so that the angular speed about the axis of the rod is w. The angular momentum of the entire assembly about the point ‘0’ is L (see the figure). Which of the following statement(s) is (are) true? a) The magnitude of the z component of Lis 55 ma*w ) The magnitude of angular momentum of centre of mass of the assembly about the point 0 is81ma’o ©) The centre of mass of the assembly rotates about the z-axis with an angular speed of «/S d) The magnitude of angular momentum of the assembly about its centre of mass is 17 ma’w/2 Question 1.18 A particle is moving ina circle x” + y” = R° with speed v,, Find angular momentum about (0,0, z) Question 1.19 For the projectile motion, calculate the torque about point 0 (point where projectile is launched) at any instant for which t < 2u sin®/g and use torque to find L. Answer: L = mgucos6t’/2(— k), t = umgcos0(— k) Question 1.20 (a) A particle of mass m moves with constant speed v along a circle of radius R. Calculate angular momentum about a point on the circumference of the circle. a) 2mvR cos"0/2 k b) mR cos’0/2 k Visit: wwwmyprepkitin or Download the PrepKit App Page S Contact us: contact@myprepkitin Whatsapp: 9971029358 IIT-JAM | TIFR | JEST | CSIR NET | GATE | CUET Physics PrepKit ) 2mvR sin’0/2 k d) 2mvR cos’0k Question 1.21 (c) A particle of mass m moves with constant speed v along a circle of radius R. Calculate torque on particle about a point on the circumference of the circle. a) mv"cos 0/2 k b) — mv’cos 0/2 k )— mv'sin0/2k d) mv’sin 0k Question 1.22 [IITJAM 2009] (a) A thin massless rod of length 21 has equal point masses m attached at its ends (see figure). The rod is rotating about an axis passing through its centre and making angle @ with it. The magnitude of the rate of . dL ' change of its angular momentum-S— is a) 2ml’w’sinOcos0 b) 2ml'w’ sind ©) 2ml’w'sin’6 d) 2ml’w*cos"0 Question 1.23 [1ISc 2007] (c) A ball tied to a massless string is winding on a cylinder which is fixed firmly to the ground, ‘The Angular momentum of the ball a) Increases with time b) Decreases with time and approaches with zero c) Remains constant 4d) Decreases and then asymptotically approaches a non-zero constant. Question 1.24 [IIT JAM 2014] (d) At an instant shown, three point masses m, 2m and 3m rest on a 3m horizontal surface, and are at the vertices of an equilateral triangle of unit side length, Assuming that G is the gravitational constant, the magnitude and direction of the torque on the mass 3m, about the point 7 2m 0, at that instant is 306 a) zero b) +G\3m’, going into the paper is 3 ©) 3G/3m°, Coming out of the paper) 4G /3m’, going into the paper Question 1.25 [IIT JAM 2016] [MSQ] (a,b) A particle moves in a circular path in the xy- plane centred at the origin. If the speed of the Visit: wwwmyprepkitin or Download the PrepKit App Page 6 Contact us: contact@myprepkitin Whatsapp: 9971029358 IIT-JAM | TIFR | JEST | CSIR NET | GATE | CUET Physics PrepKit particle is constant, then its angular momentum (a) about the origin is constant both in magnitude and direction (b) about (0,0,1) is constant in magnitude but not in direction (© about (0,0,1)varies both in magnitude and direction (@) about (0,0,1) is constant in direction but not in magnitude Question 1.26 [IIT JAM 2018] (5.28) A particle of mass m is moving in a circular orbit given by x = R cos(wt); y = R sin(wt), as observed in an inertial frame S,, Another inertial frame S, moves with uniform velocity v = Ri with respect to S,. S, and S, are related by Galilean transformation, such that the origins coincide at ¢ = 0. The magnitude of the angular momentum of the particle at t = 2x/a, as observed in S$, about its origin, is expressed as (mR’w)x. Then x is ____. (Specify your answer up to two digits after the decimal point.) Question 1.27 (a) Find the torque due to friction about the centre point of disc of radius R having constant mass density o placed on the table (1 > coefficient of friction between the disc and table) a) Byogk* b) fyogr* ©) HogR” @) Fuogr? Question 2.1 (0.24 m) A beam, balanced on a pin, has a 0,050kg mass placed 0,32m to the left of the centre of the beam. Where should a 0,067kg mass be placed, right of the centre of the beam, so that the beam stays in equilibrium? Question 2.2 (b) Find the minimum value of force for which square block of mass m and side ‘d’ will rotate about the point 0, 0 a) mg b) mg/2 c) 2mg/3 d) 2mg Visit: wwwmyprepkitin or Download the PrepKit App Page 7 Contact us: contact@myprepkitin Whatsapp: 9971029358 IIT-JAM | TIFR | JEST | CSIR NET | GATE | CUET Physics PrepKit Question 2.3 (a) Find the shift in normal when force is applied on the square block of mass m and side ‘d’ such that the block is stil at rest. a) Fd/mg b) Fd/2mg ) 2Fd/mg do Question 2.4 (b) Find the value of minimum force required to rotate the rod of mass m(constant linear mass density) placed on the table with one fixed end. 1 is the coefficient of friction between the table and rod. a) umg b) umg/2 c) 3ymg/2 d) 2nmg/3 Question 2.5 (b) Find the magnitude of the hinge force on the fixed end in the above question. a) umg b) umg/2 ) 3umg/2 d) 2umg/3 Question 2.6 (a) @ n ty A uniform rod of 20 kg is hanging in a horizontal position with the help of two threads. It supports a 40 kg mass as shown in the A B figure. Find the tensions developed in each thread. 40kg a) T, = 400 NandT, = 200N b)T, = 500 Nand, = 250N ©)T, = 200 NandT, = 400 N QT, = 250 NandT, = 500N Question 2.7 [JAM 2017] (b) The linear mass density of a rod of length L varies from one end to other as 2,(1 + x°/L"), where x is the distance from one end with tension and in them (see figure), and is a constant. The rod is suspended from a ceiling by two massless strings. Then, which of the following statements is correct ? a) The mass oftherodis 2A,L/3 T, b) The centre of gravity of the rod is located at x = 9L/16 1 2 ©) The tension in the left string is BA,Lg/12 4) The tension in the left string is 3A,L9/2 x=0 x=L Question 2.8 [TIFR 2016] (4) A uniform solid wheel of mass M and radius r is halted at a step of height h as shown in the figure. The minimum force F, applied horizontally at the centre of the wheel, necessary to raise the wheel over this step is Visit: wwwmyprepkitin or Download the PrepKit App Page 8 Contact us: contact@myprepkitin Whatsapp: 9971029358 IIT-JAM | TIFR | JEST | CSIR NET | GATE | CUET Physics PrepKit Jh@r—h Vir +h) M9 eR OM hr + hy har — hy —_—— (d) © mg MO (mg AO Question 2.9 [TIFR 2014] (b) A uniform ladder of length 2L and mass ‘m’ leans against a wall in a vertical plane at an angle 8 to the horizontal. The floor is rough, having a coefficient of static friction 41. A person of mass M stands on the ladder at a distance D from its base (see figure). If the wall is frictionless, the maximum distance (D,_,,) up the ladder that the person can reach before the ladder slips is (a) ut (1 +7) tana (o) {au (1+2) uy () wLtane (@) 2b tane Question 2.10 (a) A uniform rod is made to lean between a rough vertical wall and the ground. Given: ,, and H,, represents the coefficient of friction between the rod and wall; and the rod and the ground respectively. Find the least angle at which the rod can be leaned without slipping is aif o, “aif tone, 1 (274 1 f 2H a) 0 = tan ( Ty ) b) 6 = tan ( iS ) a1 (24H, g(a ©) @=tan 7h, 4) 0 = tan’ a Question 2.11 (c) Find the friction force acting on a rod of mass m, length L at point B if it is not sliding, as shown in the figure. a) (mg sin20)/2 b) (mg cos20)/2 ©) (ing sin20)/4 4) (mg cos20)/4 Visit: wwwinypreplitin or Download the PrepKit App Page 9 Contact us: contact@myprepkitin Whatsapp: 9971029358

You might also like